Difference between revisions of "2003 AMC 12B Problems/Problem 21"
Math Kirby (talk | contribs) m |
(→Solution 3 (Geometric Probability)) |
||
Line 44: | Line 44: | ||
~PeterDoesPhysics | ~PeterDoesPhysics | ||
+ | ==Solution 4 (Triangle Inequality)== | ||
+ | |||
+ | |||
+ | Note that we can treat <math>\text{ABC}</math> as a triangle with side lengths <math>5</math>, <math>8</math> and <math>AC=x.</math> Because <math>0</math> and <math>\pi</math> are not pat of the interval of valid <math>\alpha</math> values, <math>\triangle \text{ABC}</math> is a non-degenerate triangle. Then, by the Triangle Inequality, <math>5+8>x,</math> <math>5+x>8,</math> and <math>8+x>5.</math> These reduce to <math>x<13,</math> <math>x>3,</math> and <math>x>-3.</math> Thus, the possible values of x are <math>3<x<13,</math> or <math>x=\text{[}4,5,6,7,8,9,10,11,12\text{]}.</math> Of these <math>9</math> possible <math>x,</math> <math>3</math> of them are less than <math>7,</math> so the probability that <math>x<7</math> is <math>\frac39=\frac13=\boxed{\text{(D)}}.</math> | ||
+ | |||
+ | |||
+ | ~~AndrewZhong2012~~ | ||
== See also == | == See also == |
Latest revision as of 10:01, 8 August 2025
Contents
Problem
An object moves cm in a straight line from
to
, turns at an angle
, measured in radians and chosen at random from the interval
, and moves
cm in a straight line to
. What is the probability that
?
Solution 1 (Trigonometry)
By the Law of Cosines,
It follows that , and the probability is
.
Solution 2 (Analytic Geometry)
, let the object turn clockwise.
Let ,
.
Note that the possible points of create a semi-circle of radius
and center
. The area where
is enclosed by a circle of radius
and center
. The probability that
is
.
The function of is
, the function of
is
.
is the point that satisfies the system of equations:
,
,
,
,
Note that is a
triangle, as
,
,
. As a result
,
.
Therefore the probability that is
Solution 3 (Geometric Probability)
Setting we get that
, after assuming segment AB to be straight in the x-direction relative to our coordinate system (in other words, due to symmetrically we can set
for point B). This gives
. Using the distance formula we get
. After algebra, this simplifies to
. After evaluating the constraints of the problem, we land on option (D).
~PeterDoesPhysics
Solution 4 (Triangle Inequality)
Note that we can treat as a triangle with side lengths
,
and
Because
and
are not pat of the interval of valid
values,
is a non-degenerate triangle. Then, by the Triangle Inequality,
and
These reduce to
and
Thus, the possible values of x are
or
Of these
possible
of them are less than
so the probability that
is
~~AndrewZhong2012~~
See also
2003 AMC 12B (Problems • Answer Key • Resources) | |
Preceded by Problem 20 |
Followed by Problem 22 |
1 • 2 • 3 • 4 • 5 • 6 • 7 • 8 • 9 • 10 • 11 • 12 • 13 • 14 • 15 • 16 • 17 • 18 • 19 • 20 • 21 • 22 • 23 • 24 • 25 | |
All AMC 12 Problems and Solutions |
These problems are copyrighted © by the Mathematical Association of America, as part of the American Mathematics Competitions.